Đến nội dung

Hình ảnh

$\frac{1}{a}+\frac{1}{b}+\frac{1}{c}\geq 4(a+b+c)$


  • Please log in to reply
Chủ đề này có 6 trả lời

#1
Poseidont

Poseidont

    Dark Knight

  • Thành viên
  • 322 Bài viết
Trầm quá , một bài nhẹ để mọi người lấy lại khoan khoái.
Cho a,b,c là các số thực dương thỏa mãn
$\frac{1}{1+a}+\frac{1}{1+b}+\frac{1}{1+c}=2$
CMR,
1)$abc\leq \frac{1}{8}$ (cái này có nhiều rồi)
2) $\frac{1}{a}+\frac{1}{b}+\frac{1}{c}\geq 4(a+b+c)$
Bàu 2 nhiều cách nhé, cách của mình
Từ giả thiết $\Rightarrow \frac{a}{1+a}+\frac{b}{1+b}+\frac{c}{1+c}=1$
Đến đây đơn giản rồi, mọi người cùng thảo luận nhé

Bài viết đã được chỉnh sửa nội dung bởi Nguyen Duc Nghia: 04-08-2012 - 11:31

Nguyễn Đức Nghĩa tự hào là thành viên VMF


#2
Tru09

Tru09

    Thiếu úy

  • Thành viên
  • 625 Bài viết

Trầm quá , một bài nhẹ để mọi người lấy lại khoan khoái.
Cho a,b,c là các số thực dương thỏa mãn
$\frac{1}{1+a}+\frac{1}{1+b}+\frac{1}{1+c}=2$
CMR,
1)$abc\leq \frac{1}{8}$ (cái này có nhiều rồi)

Chém nhanh nào ^^
$GT \rightarrow \frac{1}{1+a} =\frac{b}{1+b} +\frac{c}{1+c}$
$\rightarrow \frac{1}{1+a} \geq 2\sqrt{\frac{bc}{(1+b)(1+c)}}$
Xây dựng các bDT tương tự
$\rightarrow \frac{1}{1+b} \geq 2\sqrt{\frac{ac}{(1+a)(1+c)}}$
$\rightarrow \frac{1}{1+c} \geq 2\sqrt{\frac{ab}{(1+a)(1+b)}}$
$\rightarrow \frac{1}{(1+a)(1+b)(1+c)} \geq 8.\frac{abc}{(1+a)(1+b)(1+c)}$
$\rightarrow \frac{1}{8} \geq abc \rightarrow DPCM$

Bài viết đã được chỉnh sửa nội dung bởi Tru09: 04-08-2012 - 11:25


#3
bastian schweinsteiger

bastian schweinsteiger

    Hạ sĩ

  • Thành viên
  • 74 Bài viết

Trầm quá , một bài nhẹ để mọi người lấy lại khoan khoái.
Cho a,b,c là các số thực dương thỏa mãn
$\frac{1}{1+a}+\frac{1}{1+b}+\frac{1}{1+c}=2$
CMR,
1)$abc\leq \frac{1}{8}$ (cái này có nhiều rồi)
2) $\frac{1}{a}+\frac{1}{b}+\frac{1}{c}\geq 4(a+b+c)$

$\frac{1}{a}+\frac{1}{b}+\frac{1}{c}\geq 4(a+b+c)\Leftrightarrow ab+bc+ca\geq 4abc(a+b+c)$
mà $abc\leq \frac{1}{8}$
ta cần CM $2(ab+bc+ca)\geq a+b+c\Leftrightarrow 2(\frac{1}{a}+\frac{1}{b}+\frac{1}{c})\geq \frac{1}{ab}+\frac{1}{bc}+\frac{1}{ca}$ (1)
mà $2=\frac{1}{1+a}+\frac{1}{1+b}+\frac{1}{1+c}\Rightarrow 1=\frac{1}{1+1+2a}+\frac{1}{1+1+2b}+\frac{1}{1+1+2c}\leq \frac{1}{9}(2+\frac{1}{2a}+2+\frac{1}{2b}+2+\frac{1}{2c})=\frac{2}{3}+\frac{1}{18}(\frac{1}{a}+\frac{1}{b}+\frac{1}{c})\Rightarrow 18\leq 12+\frac{1}{a}+\frac{1}{b}+\frac{1}{c}\Rightarrow \frac{1}{a}+\frac{1}{b}+\frac{1}{c}\geq 6$
$\Rightarrow (1)\Leftrightarrow (\frac{1}{a}+\frac{1}{b}+\frac{1}{c})^{2}\geq \frac{1}{3}(\frac{1}{ab}+\frac{1}{bc}+\frac{1}{ca})\Rightarrow Q.E.D$

#4
dark templar

dark templar

    Kael-Invoker

  • Hiệp sỹ
  • 3788 Bài viết

Cho a,b,c là các số thực dương thỏa mãn
$\frac{1}{1+a}+\frac{1}{1+b}+\frac{1}{1+c}=2$
2) $\frac{1}{a}+\frac{1}{b}+\frac{1}{c}\geq 4(a+b+c)$

Làm chặt một xíu nhé :D
Chứng minh:
$$\frac{1}{a}+\frac{1}{b}+\frac{1}{c} \ge 4(a+b+c)+\frac{(2a-1)^2}{(a+2010)(2012a+2011)}$$
Trong đó $a=\max \{a;b;c \}$.
"Do you still... believe in me ?" Sarah Kerrigan asked Jim Raynor - Starcraft II:Heart Of The Swarm.

#5
WhjteShadow

WhjteShadow

    Thượng úy

  • Phó Quản lý Toán Ứng dụ
  • 1323 Bài viết
Cách khác ch0 bài 2:
Do $\frac{1}{1+a}+\frac{1}{1+b}+\frac{1}{1+c}=2$ Nên ta có thể đặt $a=\frac{x}{y+z},b=\frac{y}{x+z},c=\frac{z}{x+y}$
Lúc đó ta cần chứng minh:
$$\frac{x+y}{z}+\frac{x+z}{y}+\frac{y+z}{x}\geq 4(\frac{x}{y+z}+\frac{y}{x+z}+\frac{z}{x+y})$$
Nhưng điều này đúng khi ta cộng vế the0 vế 3 bất đẳng thức cơ bản:
$\frac{x}{y}+\frac{x}{z}\geq 4\frac{x}{y+z}$
$\frac{y}{z}+\frac{y}{x}\geq 4\frac{y}{x+z}$
$\frac{z}{x}+\frac{z}{y}\geq 4\frac{z}{x+y}$
CÒn bài mở rộng của a Phúc thì:
$$Q.e.D\Leftrightarrow \sum (\frac{x}{y}+\frac{x}{z}- 4\frac{x}{y+z})\geq \frac{(2x-y-z)^2}{[x+2010(y+z)][2012x+2011(y+z)]}$$
$$\Leftrightarrow \sum [\frac{x(y-z)^2}{yz(y+z)}]\geq \frac{(2x-y-z)^2}{[x+2010(y+z)][2012x+2011(y+z)]}$$
X0ng rồi đến hay mềnh $Cauchy-Schwar$ hay $S.O.S$ nhể.A Phúc ơi a thử xem lại đề hộ em với em $S.O.S$ đau cả tay mãi k ra T.T

Bài viết đã được chỉnh sửa nội dung bởi WhjteShadow: 04-08-2012 - 23:40

“There is no way home, home is the way.” - Thich Nhat Hanh

#6
dark templar

dark templar

    Kael-Invoker

  • Hiệp sỹ
  • 3788 Bài viết

CÒn bài mở rộng của a Phúc thì:
$$Q.e.D\Leftrightarrow \sum (\frac{x}{y}+\frac{x}{z}- 4\frac{x}{y+z})\geq \frac{(2x-y-z)^2}{[x+2010(y+z)][2012x+2011(y+z)]}$$
$$\Leftrightarrow \sum [\frac{x(y-z)^2}{yz(y+z)}]\geq \frac{(2x-y-z)^2}{[x+2010(y+z)][2012x+2011(y+z)]}$$
X0ng rồi đến hay mềnh $Cauchy-Schwar$ hay $S.O.S$ nhể.A Phúc ơi a thử xem lại đề hộ em với em $S.O.S$ đau cả tay mãi k ra T.T

Bài này không sai đâu :D Cái số 2010;2011;2012 chỉ để hù em mà thôi =))
P/s:SOS hay SOC không "giết" được bài này ;) Nhớ để ý là $a=\max \{a;b;c \}$

Bài viết đã được chỉnh sửa nội dung bởi dark templar: 05-08-2012 - 21:11

"Do you still... believe in me ?" Sarah Kerrigan asked Jim Raynor - Starcraft II:Heart Of The Swarm.

#7
WhjteShadow

WhjteShadow

    Thượng úy

  • Phó Quản lý Toán Ứng dụ
  • 1323 Bài viết
Ta sẽ chứng minh 1 bài toán mạnh hơn bài toán của a Phúc:Với $\frac{1}{a+1}+\frac{1}{b+1}+\frac{1}{c+1}=1$,$a=max(a;b;c)$ thì:
$$\frac{1}{a}+\frac{1}{b}+\frac{1}{c}\geq 4(a+b+c)+\frac{(2a-1)^2}{a(2a+1)}$$
Giải:
Đổi biến như trên và ta sẽ chứng minh 2 điều sau:
$b+c\geq \frac{2}{2a+1}$ (1)
Và $bc \leq \frac{1}{(2a+1)^2}$ (2)
Thật vậy $(1)\Leftrightarrow \frac{y}{x+z}+\frac{z}{x+y}\geq \frac{2(y+z)}{2x+y+z}$
Mà the0 $Cauchy-Schwarz$ thì $\frac{y}{x+z}+\frac{z}{x+y}\geq \frac{(y+z)^2}{xy+2yz+zx}$
Nên ta chỉ cần chứng minh: $\frac{(y+z)^2}{xy+2yz+zx}\geq \frac{2(y+z)}{2x+y+z}$
$\Leftrightarrow (y+z)(2x+y+z)\geq 2(xy+2yz+zx)$
$\Leftrightarrow (y-z)^2\geq 0$ (Luôn đúng) $\to$ (1) được chứng minh.
CÒn $(2)\Leftrightarrow \frac{yz}{(x+y)(x+z)}\leq \frac{(y+z)^2}{(2x+y+z)^2}$
$\Leftrightarrow (y+z)^2(x+y)(x+z)\geq yz(2x+y+z)^2$
$\Leftrightarrow x^2y^2-2x^2yz+x^2z^2+x[y^3+z^3-yz(y+z)]\geq 0$
$\Leftrightarrow x^2(y-z)^2+x[y^3+z^3-yz(y+z)]\geq 0$ (Luôn đúng)
Vậy từ (1) và (2) ta có:
$\frac{1}{b}+\frac{1}{c}-4(b+c)=(b+c)(\frac{1}{bc}-4)\geq \frac{2[(2a+1)^2-4]}{2a+1}=4a-\frac{1}{a}+\frac{(2a-1)^2}{a(2a+1)}$

(Do $abc\leq \frac{1}{8}$ mà $a=max(a;b;c)$ Nên $ab\leq \frac{1}{4}$ hay $4-\frac{1}{ab}\geq 0$)
$\Leftrightarrow \frac{1}{a}+\frac{1}{b}+\frac{1}{c}\geq 4(a+b+c)+\frac{(2a-1)^2}{a(2a+1)}$
Vậy ta có điều phải chứng minh.Dâu bằng xảy ra khi $a=b=c=\frac{1}{2}$

Bài viết đã được chỉnh sửa nội dung bởi WhjteShadow: 06-08-2012 - 12:03

“There is no way home, home is the way.” - Thich Nhat Hanh




1 người đang xem chủ đề

0 thành viên, 1 khách, 0 thành viên ẩn danh